next up previous
Next: About this document Up: No Title Previous: Solution to Problem 2.

Solution to Problem 3.

3. [40 points] In each of the examples below, a sequence tex2html_wrap_inline736 , tex2html_wrap_inline738 of random variable is defined. For each example, determine if it converges in any of the modes we have discussed in class:

(i)
almost surely,
(ii)
in probability,
(iii)
in r'th mean for some r > 0,
(iv)
in distribution.

If it does converge, describe the limiting random variable or distribution. Justify your answers.

(a) [20 points] tex2html_wrap_inline736 = tex2html_wrap_inline748 where U is a random variable which is uniformly distributed on (0,1), and tex2html_wrap_inline754 denotes the indicator function of the open interval (0,1/n).

Solution: Clearly, tex2html_wrap_inline758 = 1/n, so tex2html_wrap_inline764 . Since convergence in probability implies convergence in distribution, we know tex2html_wrap_inline766 . In fact, it is clear that tex2html_wrap_inline768 . To see, note that on the event [ U > 0 ] (which has probability 1), that for all n > 1/U we have tex2html_wrap_inline776 . Thus, with probability 1 the sequence of random variables is ``eventually'' 0 (i.e., from some point on, the sequence is zero, but the point where this occurs is random).

Now as far as convergence in r'th mean, we have

  equation299

The latter limit holds since the exponential function tex2html_wrap_inline784 goes to infinity faster than the linear function n. Hence, tex2html_wrap_inline736 does not converge in r'th mean to 0 for any r > 0.

REMARK: Some students observed that

displaymath732

and invoked the second Borel Cantelli Lemma to claim the sequence does NOT converge a.s. What is wrong with this argument? The second B.C. lemma requires independence, and these random variables are highly dependent.

(b) [20 points]

displaymath733

where Z is a N(0,1) random variable, U is uniformly distributed on (0,1), and tex2html_wrap_inline804 = tex2html_wrap_inline808 is the sample mean of i.i.d. random variable tex2html_wrap_inline810 having an exponential distribution with mean 1.

Solution: We claim that tex2html_wrap_inline814 . Now tex2html_wrap_inline816 because each tex2html_wrap_inline818 is N(0,1). Also, tex2html_wrap_inline822 (in fact, tex2html_wrap_inline824 so tex2html_wrap_inline826 ). Also, by the Weak Law of Large numbers, tex2html_wrap_inline828 . So, by Slutsky's theorem, tex2html_wrap_inline830 tex2html_wrap_inline698 Z, and also by Slutsky's theorem, tex2html_wrap_inline836 tex2html_wrap_inline698 Z/1 = Z.

We claim tex2html_wrap_inline736 does not converge in probability. Clearly, for large n, tex2html_wrap_inline736 is approximately equal to tex2html_wrap_inline852 because of the alternating signs of tex2html_wrap_inline818 which is the ``dominant'' term in tex2html_wrap_inline736 . We will show essentially that the sequence violates a Cauchy condition for convergence in probability. Assume that tex2html_wrap_inline858 (where of course X would have to have a N(0,1) distribution by the last paragraph and the fact that tex2html_wrap_inline864 implies tex2html_wrap_inline698 ). Then we have for any tex2html_wrap_inline654 ,

   eqnarray324

Now

      eqnarray331

Put otherwise,

  equation344

Now

    eqnarray348

where

    eqnarray362

Now we claim each tex2html_wrap_inline870 = tex2html_wrap_inline874 as tex2html_wrap_inline626 . Assuming this is the case we would have in conjunction with (26) that

  equation374

by a homework exercise, but

  equation381

and the latter has a N(0,4) distribution for all n and so clearly does not converge to 0 in distribution and hence not also in probability. Thus, we have obtained a contradiction, and hence we do not have convergence in probability.

There remains to verify each tex2html_wrap_inline884 . For tex2html_wrap_inline886 , we have tex2html_wrap_inline888 and by a continuous mapping principle, tex2html_wrap_inline890 , and hence also tex2html_wrap_inline892 . Thus, by continuous mapping again we have tex2html_wrap_inline894 = tex2html_wrap_inline874 . Now tex2html_wrap_inline818 = tex2html_wrap_inline904 , so tex2html_wrap_inline886 = tex2html_wrap_inline910 = tex2html_wrap_inline874 by the infamous homework exercise. Similarly tex2html_wrap_inline916 = tex2html_wrap_inline920 = tex2html_wrap_inline924 = tex2html_wrap_inline874 , and similarly for tex2html_wrap_inline930 .

Since tex2html_wrap_inline736 fails to converge in probability, it also cannot converge almost surely or in r'th mean since convergence in either of these modes would imply convergence in probability.


next up previous
Next: About this document Up: No Title Previous: Solution to Problem 2.

Dennis Cox
Tue Nov 11 20:55:50 CST 1997